Compare the power of one motorcylce that travels twice as fast as a second indetical motorcycle.A.Same amount of powerB.Twice the powerC.4 times the powerD.Half the power

Answers

Answer 1

Given:

The power of one motorcycle travels twice as fast as a second motorcycle.

To compare the power of motorcycles.

Explanation:

Let the speed of one motorcycle be v.

The speed of the second motorcycle will be 2v.

Power is calculated by the formula

[tex]P=\text{ Force}\times velocity[/tex]

The power of the first motorcycle will be

[tex]P_1=Fv_1[/tex]

The power of the second motorcycle will be

[tex]\begin{gathered} P_2=Fv_2 \\ =F\times2v_1 \\ =2Fv_1 \\ =2P_1 \end{gathered}[/tex]

Thus, the power of the second motorcycle is twice the power of the first motorcycle.


Related Questions

A 10 gram ball is rolling at 3 m/s. Calculate its kinetic energy.

Answers

ANSWER:

0.045 joules

STEP-BY-STEP EXPLANATION:

Given:

mass (m) = 10 g = 0.01 kg

velocity (v) = 3 m/s

The kinetic energy is given by the following formula:

[tex]K_E=\frac{1}{2}mv^2[/tex]

We replacing:

[tex]\begin{gathered} K_E=\frac{1}{2}\cdot0.01\cdot3^2 \\ K_E=0.045\text{ J} \end{gathered}[/tex]

The kinetic energy is 0.045 joules.

Describe in simple terms each of Kepler's three laws. Why were Tycho Brahe's observations so important for Kepler to develop these laws?

Answers

Kepler's first law of planetary motion states that every planet’s orbit is elliptical and has the Sun at a focus

Kepler's second law of planetary motion states that the line joining the Sun and a planet sweeps out equal areas in equal times

Kepler's third law of planetary motion states that the square of a planet’s orbital period is proportional to the cube of the semi-major axis of its orbit

Tycho Brahe provided some accurate astronomical data which makes it possible for Kepler to develop the laws of planetary motion. He devised the best instruments available before the invention of the telescope. These data and instrument provided by Tycho Brahe was able to assist Kepler in formulating these laws

If a rock has 376 J of potential energy when it’s held 10.1 m above the ground what is its mass? Round to the nearest tenth

Answers

Answer: 3.8 kg

Explanation:

The formula for calculating potential energy is expressed as

Potential energy = mgh

where

m is the mass of the object

g is the acceleration due to gravity

h is the height above the ground

From the information given,

Potential energy = 376

g = 9.8 m/s^2

h = 10.1

Thus,

376 = m x 9.81 x 10.1

376 = 99.081m

m = 376/99.081

m = 3.8 kg

The mass is 3.8 kg

Two astronauts, of masses 60 kg and 80 kg, are initially right next to each other and at rest in outer space. They suddenly push each other apart. What is their separation after the heavier astronaut has moved 12 m?24 m16 m9.0 m28 m21 m

Answers

We will have the following:

First, we recall that momentum is given by:

[tex]m_1v_0+m_2v_0=m_1v_1+m_2v_2[/tex]

So:

[tex]\begin{gathered} (60kg)(0m/s)+(80kg)(0m/s)=(60kg)v_1+(80kg)v_2 \\ \\ \Rightarrow v_1=\frac{-(80kg)v_2}{(60kg)}\Rightarrow v_1=-\frac{4}{3}v_2 \end{gathered}[/tex]

Then:

[Let's assume that when astronaut 2 moved 12 meters = t]

[tex]\begin{gathered} x_1=v_1t\Rightarrow x_1=-\frac{4}{3}v_2t \\ \\ x_2=v_2t\Rightarrow t=\frac{12}{v_2} \end{gathered}[/tex]

Then:

[tex]x_1=-\frac{4}{3}v_2\ast\frac{12}{v_2}\Rightarrow x_1=-16m[/tex]

Finally, the total distance will be:

[tex]\begin{gathered} d=|x_1|+|x_2|\Rightarrow d=16m+12m \\ \\ \Rightarrow d=28m \end{gathered}[/tex]

A diagram of a student on a playground swing is shown below. At which point is the kinetic energy the greatest?A. Point 1B. Point 2C. Point 3D. Point 4

Answers

To find:

The point where the kinetic energy is the greatest.

Explanation:

The kinetic energy of an object is the energy possessed by the object due to its motion. Thus the kinetic energy of an object is proportional to the square of the velocity of the object.

The velocity of the wing will be zero at the extreme points of the path of the swing. But as the swing approaches the mean point, i.e., point 3, the velocity of the swing increases. The velocity will be maximum at the mean point.

Thus the kinetic energy will be the greatest at point 3.

Final answer:

Therefore the correct answer is option C.

A concave mirror of focal length 10 cm forms an upright and diminished image of a real object placed at a distance of 5 cm from the mirror. Is this true or false?

Answers

Given,

The focal length of the concave mirror, f=10 cm

The object distance, o=5 cm

From the mirror formula, we have,

[tex]\frac{1}{f}=\frac{1}{o}+\frac{1}{i}[/tex]

Where i is the image distance.

On substituting the known formula,

[tex]\begin{gathered} \frac{1}{10}=\frac{1}{5}+\frac{1}{i} \\ \Rightarrow\frac{1}{i}=\frac{1}{10}-\frac{1}{5} \\ =-\frac{1}{10} \\ \Rightarrow i=-10\text{ cm} \end{gathered}[/tex]

And the magnification is given by,

[tex]m=\frac{-i}{o}[/tex]

On substituting the known values,

[tex]\begin{gathered} m=\frac{-(-10)}{5} \\ =2 \end{gathered}[/tex]

Thus the object is not diminished, it is magnified.

Thus the given statement is false.

If you have a convex lens whose focal length is 10.0 cm, where would you place an object in order to produce an image that is virtual?

Answers

When an object is placed between first focus and optical center of a convex lens then virtual image is produced.

Here , focal length is 10.0 cm . So object distance should be less than 10.0 cm

Final answer is : between focus and optical center of the lens

I need help with #2 it off s for practice

Answers

First, find the acceleration using Newton's Second Law.

[tex]\begin{gathered} F=ma \\ a=\frac{F}{m} \end{gathered}[/tex]

Where F = 8.10x10^5 N and m = 1.40x10^7 kg.

[tex]a=\frac{8.10\times10^5N}{1.40\times10^7\operatorname{kg}}=5.79\times10^{-2}\cdot\frac{m}{s^2}[/tex]

Then, use a formula that relates acceleration, initial velocity, final velocity, and time.

[tex]v_f=v_0+at[/tex]

Solve for t because the problem is asking to find the time.

[tex]\begin{gathered} v_f-v_0=at_{} \\ t=\frac{v_f-v_0}{a} \end{gathered}[/tex]

Where vf = 64 km/h, v0 = 0, and a = 5.79x10^-2 m/s^2. Before we continue, we need to transform the final velocity to m/s.

[tex]v_f=64\cdot\frac{km}{h}\cdot\frac{1000m}{1\operatorname{km}}\cdot\frac{1h}{3600\sec}=17.78\cdot\frac{m}{s}[/tex]

Once we have the velocity transformed, we are able to find t.

[tex]\begin{gathered} t=\frac{17.78\cdot\frac{m}{s}-0}{5.79\times10^{-2}\cdot\frac{m}{s^2}} \\ t=3.07\times10^2\sec \\ t=307\sec \end{gathered}[/tex]

But, the answer must be in minutes.

[tex]t=307\sec \cdot\frac{1\min}{60\sec}=5.12\min [/tex]

Therefore, it takes 5.12 minutes.

I was told it was 5.886 J by another tutor on here but that was incorrect so just trying again

Answers

The potential energy of gravity is given by:

[tex]\begin{gathered} E=mgh \\ where: \\ m=0.3 \\ h=2 \\ g=9.8 \\ so: \\ E=0.3\cdot2\cdot9.8 \\ E\approx5.9J \end{gathered}[/tex]

Answer:

5.9 J

If the man and woman are taken to a planet where the acceleration due to gravity is twice that of earth repeat the woman mass was 25kg on earth and the man was 300N on another planet

Answers

The mass of the woman is 25 kg because the mass is constant.

The mass of the man can be found using the formula: W = mg, where g is double Earth's gravity.

[tex]\begin{gathered} m=\frac{W}{g} \\ m=\frac{300N}{2\cdot9.8\cdot\frac{m}{s^2}} \\ m=\frac{300}{19.6}kg \\ m\approx15.3\operatorname{kg} \end{gathered}[/tex]

The mass of the man is 15.3 kg.

The weight of the man on Earth can be found with the same formula but using Earth's gravity.

[tex]\begin{gathered} W=15.3\operatorname{kg}\cdot9.8\cdot\frac{m}{s^2} \\ W=149.94N \end{gathered}[/tex]

The weight of the man on Earth is 149.94 N.

At last, the weight of the woman on Earth can be found using the same method before.

[tex]\begin{gathered} W=25\operatorname{kg}\cdot9.8\cdot\frac{m}{s^2} \\ W=245N \end{gathered}[/tex]

The weight of the woman on Earth is 245N.

Question 28 of 30The graph below shows the conservation of energy for a skydiver jumping outof a plane and landing safely on the ground. Which energy is represented byline A?сAEnergy (kJ)BDistance (m)A. Potential energyB. Kinetic energy

Answers

Answer:

Explanation:

Potential energy depends on the height of the object above the ground. As the height decreases, the potential energy decreases. Looking at line A, the energy is decreasing as the distance is increasing. Thus, the energy represented by line A is

A. Potential energy

A deuteron particle consists of one proton and one neutron and has a mass of 3.34x10^-27 kg. A deuteron particle moving horizontally enters uniform, vertical 0.0800 T magnetic field and follows a circular arc of radius of 38.5 cm. What would be the radius of the arc followed by a proton that entered the field with the same velocity as the deuterium?

Answers

_dThe radius of curvature of a subatomic particle under a magnetic field is given by the following formula:

[tex]r=\frac{mv}{qB}[/tex]

Where:

[tex]\begin{gathered} r=\text{ radius} \\ v=\text{ velocity} \\ q=\text{ charge} \\ B=\text{ magnetic field} \end{gathered}[/tex]

We can determine the quotient between the velocity and the charge of the deuteron particle from the formula. First, we divide both sides by the mass:

[tex]\frac{r_d}{m_d}=\frac{v}{q_B_}[/tex]

Now, we multiply both sides by the magnetic field "B":

[tex]\frac{Br_d}{m_d}=\frac{v}{q}[/tex]

Since the charge of the deuterion is the same as the charge of the proton and the velocity we are considering are the same this means that the quotient between velocity and charge is the same for both particles. Therefore, we can apply the formula for the radius again, this time for the proton:

[tex]r_p=\frac{m_pv}{qB}[/tex]

And substitute the quotient between velocity and charge:

[tex]r_p=\frac{m_p}{B}(\frac{Br_d}{m_d})[/tex]

Now, we cancel out the magnetic field:

[tex]r_p=\frac{m_pr_d}{m_d}[/tex]

Now, we substitute the values:

[tex]r_p=\frac{(1.67\times10^{-27}kg)(0.385m)}{(3.34\times10^{-27}kg)}[/tex]

Solving the operations:

[tex]r_p=0.193m=19.3cm[/tex]

Therefore, the radius is 19.3 cm.

For an object starting from rest and accelerating with constantacceleration, distance traveled is proportional to the square of thetime. If an object travels 2.0 furlongs in the first 2.0 s, how far willit travel in the first 4.0 s?

Answers

Since the object is accelerating with constant acceleration we can use the following formula for the position of the object:

[tex]x=x_0+v_0t+\frac{1}{2}at^2[/tex]

where x0 is the initial position, v0 is the initial velocity, a is the acceleration and t is the time. In this case, the initial position and velocity are zero. Plugging the values given we have:

[tex]\begin{gathered} 2=\frac{1}{2}a(2)^2 \\ 2=2a \\ a=1 \end{gathered}[/tex]

Hence, the acceleration of the object is 1 furlong per second per second.

Once we know the acceleration we can use the same formula to determine how far the object will travel in four seconds.

[tex]\begin{gathered} x=\frac{1}{2}(1)(4)^2 \\ x=\frac{16}{2} \\ x=8 \end{gathered}[/tex]

Therefore, the object will travel 8 furlongs in four seconds.

a given mass of a gas at - 73°C exerts pressure of 40 cm of mercury. what pressure will be exerted at 127 °C if the volume remains constant

Answers

The pressure that will be exerted at 127 °C if the volume remains constant is 80 cm of mercury

How do I determine the new pressure?

First, we shall list out the given parameters from the question. This is shown below:

Initial temperature (T₁) = -73 °C = -73 + 273 = 200 K Initial pressure (P₁) = 40 cm of mercuryNew temperature (T₂) = 127 °C = 127 + 273 = 400 KVolume = ConstantNew pressure (P₂) = ?

Thus, we can obtain the new pressure of the gas at 127 °C as illustrated below:

P₁ / T₁ = P₂ / T₂

40 / 200 = P₂ / 400

Cross multiply

200 × P₂ = 40 × 400

200 × P₂ = 16000

Divide both sides by 200

P₂ = 16000 / 200

P₂ = 80 cm of mercury

Therefore, the new pressure is 80 cm of mercury

Learn more about gas laws:

https://brainly.com/question/15343985

#SPJ1

i need help asap i just need the vocab i need help with this please help

Answers

ANSWERS

0. First law of Thermodynamics:, ,B,. Thermal energy can change form and location, but it cannot be created or destroyed

,

1. Thermal Energy: ,H,. Kinetic energy in transit from one object to another due to temperature difference

,

2. Temperature: ,D,. the average kinetic energy of particles in an object - not the total amount of kinetic energy particles

,

3. Absolute zero: ,J, occurs when all kinetic energy is removed from an object

,

4. Thermal Equilibrium: ,E, obtained when touching objects within a system reach the same temperature

,

5. Zeroth Law of Thermodynamics: ,C, if two systems are separately found to be in thermal equilibrium with a third system, the first two systems are in thermal equilibrium with each other.

,

6. Conduction: ,F, ,the transfer of thermal energy within an object or between objects from molecule to molecule

,

7. Coefficient of Heat Conductivity: ,G, is the measure of a material's ability to conduct heat.

,

8. Thermal resistance of a material: ,A, the measure of a material's ability to resist heat

,

9. Radiation: ,I, the process by which energy is transmitted through a medium, including empty space, as electromagnetic waves.

EXPLANATION

The zeroth and first laws of thermodynamics are known statements, as well as the definition of thermal equilibrium, so there is nothing to explain about these three.

Thermal energy is a form of kinetic energy. It is "generated" by the vibrations of the molecules or atoms that form the object, causing the object to increase its temperature. As a result, the temperature is the average kinetic energy of these particles.

Absolute zero is a temperature of 0 degrees Kelvin - approximately -273 degrees Celsius. When an object reaches this particular temperature it is considered as if the object has "no temperature" - that is why it is called absolute zero. As explained above, the temperature is the average kinetic energy of the object's particles, so if the particles have no kinetic energy, then the object has zero temperature.

Heat conductivity and thermal resistance are basically opposites. The first is the object's ability to conduct heat while the second is the ability to resist the conduction of heat.

Radiation and Conduction are two ways energy can be transferred. The first does not need a medium - that is why energy can be transmitted through empty space, and the second is by contact between two objects or an object and a medium.

A human heart found to beat seventy five times in a minute. Calculate the beat frequency?​

Answers

[tex]{ \green{ \tt{f = \frac{number \: of \: beats}{time \: taken}}}} [/tex]

[tex]{ \green{ \tt{number \: of \: beats = 75}}}[/tex]

[tex]{ \green{ \tt{time \: taken =1 \: min \: = 60 \: sec}}}[/tex]

[tex]{ \red{ \sf{f = \frac{ \cancel{75^{3}}}{ \cancel{ 60_{4} }}}}}[/tex]

[tex]{ \blue{ \boxed{ \purple{ \sf{f = \frac{3}{4} = 1.2 {s}^{ - 1}}}}}} [/tex]

___________________________________

[tex]{ \blue{ \sf{T = \frac{1}{f}}}} [/tex]

[tex]{ \blue{ \sf{T = \frac{1}{ \purple{ \sf{1.2}}}}}} [/tex]

[tex]{ \boxed{ \red{ \sf{T = 0.8 \: S}}}}[/tex]

You yell down a very deep well and it takes 1.5 s for your echo to return. If the speed of sound is 340 m/s then how deep is the well ?

Answers

We know that

• The time of the echo to return is 1.5 seconds.

,

• The speed of the sound is 340 m/s.

It's important to consider that the sound wave has a constant speed, that is, it doesn't change its velocity. Therefore, we have to use the relation

[tex]d=v\cdot t[/tex]

Where t = 1.5 sec and v = 340 m/s. Let's find d

[tex]\begin{gathered} d=340m/s\cdot1.5\sec \\ d=510m \end{gathered}[/tex]Hence, the well is 510 meters deep.

12000 inches to yards

Answers

ANSWER

[tex]\begin{equation*} 333.33\text{ yds} \end{equation*}[/tex]

EXPLANATION

We want to convert 12000 inches to yards.

To do this, divide the value in inches by 36:

[tex]\begin{gathered} 1\text{ in }=\frac{1}{36}\text{ yd} \\ \\ 12000\text{ in }=\frac{12000}{36}\text{ yds }=333.33\text{ yds} \end{gathered}[/tex]

That is the answer.

A can sits on a vertical wooden fencepost 1.9 meters above the ground. Billy picks up a small rock, aims at an angle ϴ = 25⁰ above the horizontal and throws the rock, releasing it 1 m above the ground with an initial speed of v0 =10 m/s. Boom! He hits the can! How far away is the fencepost?

Answers

Given,

Height of the fencepost, h=1.9 m

Angle at which the rock was thrown, θ=25°

The height at which the rock was released, a=1 m

The initial speed of the rock, v₀=10 m/s

Referring to the diagram,

[tex]\tan \theta=\frac{h-a}{d}[/tex]

On rearranging the above equation,

[tex]d=\frac{h-a}{\tan \theta}[/tex]

On substituting the known values,

[tex]d=\frac{1.9-1}{\tan 25^0}=1.93\text{ m}[/tex]

Therefore the fencepost is at a distance of 1.93 m

URGENT!! ILL GIVE
BRAINLIEST!!!! AND 100 POINTS!!!!!!

A feather and a bowling ball are each dropped from an equal height in a vacuum and land at the same time. Which graph shows the total mechanical energy of the bowling ball as it falls?

Answers

The total mechanical energy of the bowling ball and the feather is shown by the graph in option D

What is the total mechanical energy?

We know that the mechanical energy is the energy that is possessed by a body by virtue of its motion or by virtue of its staying at a place. Thus mechanical energy is possessed by an object that is moving or by an object that is at rest.

In this case, we have a  feather and a bowling ball are each dropped from an equal height in a vacuum and land at the same time. We know that the mechanical energy of the two objects must be constant. This is because, the potential energy of the feathers and the ball at a height is converted to kinetic energy as the two objects begin to move.

The graph that would show the total energy must be one in which the energy axis of the graph is constant as we see in option D.

Learn  more about mechanical energy:https://brainly.com/question/13552918

#SPJ1

Answer:

the answer is D

Explanation:

A solenoid is wound with 259 turns per cm. An outer layer of insulated wire with 51 turns per cm is wound over the first layer of wire. The inner coil carries a current of 7.577 A, and the outer coil carries a current of 21.68 A in the opposite direction. What is the magnitude of the magnetic field, in microTeslas, at the central axis ?

Answers

Answer tab

For this question, we'll first define a number, called the "linear density of the coil" which is exactly the one the exercise gives us, in a unit of turns per length. The magnetic field can be calculated as:

[tex]B=\mu_0in[/tex]

Where n is the linear density.

In our case, as the coils carry current in opposite directions, the generated magnetic fields will be opposed, and we'll have:

[tex]B=B_{outer}-B_{inner}=\mu_0*(51*10^2*21.68-259*10^2*7.577)[/tex]

Please note that we had to multiply by 10^2, in order to convert turns/cm to turns/m

Then, our final magnetic field will be:

[tex]|B|=0.107664T=107664\mu T[/tex]

Our final answer is B=107664uT

A radioactive tracer has a half-life of two hours how much of a 2500 g sample will be available after 18 hours?

Answers

ANSWER:

4.88 grams

STEP-BY-STEP EXPLANATION:

We must first calculate how many half-life there are in 18 hours, knowing that each half-life takes 2 hours.

[tex]\frac{18}{2}=9\text{ half-life}[/tex]

Now, knowing this, we can calculate the number of grams applying 9 times the half-life, like this:

[tex]\frac{2500}{2^9}=4.88\text{ g}[/tex]

Which means that after 18 hours there are 4.88 grams

A truck can travel at 100 km/hr How long would it take to drive 900km?

Answers

Given:

The speed of the truck is,

[tex]v=100\text{ km/hr}[/tex]

The distance is,

[tex]s=900\text{ km}[/tex]

The time to drive this distance is,

[tex]t=\frac{s}{d}[/tex]

Substituting the values we get,

[tex]\begin{gathered} t=\frac{900}{100} \\ =9\text{ hrs} \end{gathered}[/tex]

Hence, the time is 9 hrs.

Objects with masses M1=12.0 kg and M2= 7.0 kg are connected by a light string that passes over a frictionless pulley as in the figure below. If, when the system starts from rest, M2 falls 1.00m in 1.33s, determine the coefficient of Kinect friction between M1 and the table

Answers

We are asked to determine the coefficient of friction in the sysmtem. First we will do a free body diagram of the first mass, like this:

Where:

[tex]\begin{gathered} N=\text{ Normal Force} \\ m_1=\text{ mass} \\ g=\text{ acceleration of gravity} \\ T=\text{ tension} \\ F_f=\text{friction force} \end{gathered}[/tex]

Now, we add the forces in the horizontal direction, we get:

[tex]T-F_f=m_1a[/tex]

Now, to determine the friction force we need to use the following relationship:

[tex]F_f=\mu N[/tex]

To determine the normal force we will add the forces in the vertical direction. Since there is no movement in the vertical direction this sum must add up to zero:

[tex]\begin{gathered} N-m_1g=0 \\ N=m_1g \end{gathered}[/tex]

Now, we substitute the value of the normal force in the equation for the friction force:

[tex]F_f=\mu m_1g[/tex]

Now, we substitute the friction force in the sum of horizontal forces:

[tex]T-\mu m_1g=m_1a[/tex]

Now, we turn our attention to the second mass. We add the forces in the vertical direction:

[tex]m_2g-T=m_2a[/tex]

Now, since the acceleration "a" and the tension "T" is the same for both masses we will solve fot "T" in the sum of forces for the second mass, like this:

[tex]m_2g-m_2a=T[/tex]

Now, we substitute the value in the sum of forces of the first mass, we get:

[tex]m_2g-m_2a-\mu m_1g=m_1a[/tex]

Now, we solve for the coefficient of friction. To do that we will subtract "m2g" to both sides:

[tex]-m_2a-\mu m_1g=m_1a-m_2g[/tex]

Now, we add "m2a" to both sides:

[tex]-\mu m_1g=m_1a-m_2g+m_2a[/tex]

Now, we divide both sides by "-m1g":

[tex]\mu=\frac{m_1a-m_2g+m_2a}{-m_1g}[/tex]

We have gotten an expression for the coefficient of friction but we need to determine the acceleration of the system. To do that we will use the fact that the second mass moves 1 meter in 1.33 seconds. Assuming constant acceleration we can use the following equation of motion:

[tex]y=v_0t+\frac{1}{2}at^2[/tex]

Where:

[tex]\begin{gathered} y=\text{ distance} \\ v_0=\text{ initial velocity} \\ t=\text{ time} \\ a=\text{ acceleration} \end{gathered}[/tex]

Since the mass starts from rest we have that the initial velocity is zero, therefore:

[tex]y=\frac{1}{2}at^2[/tex]

Now, we solve for the acceleration. First, we multiply both sides by 2:

[tex]2y=at^2[/tex]

Now, we divide both sides by the time squared:

[tex]\frac{2y}{t^2}=a[/tex]

Now, we plug in the values:

[tex]\frac{2(1m)}{(1.33s)^2}=a[/tex]

Solving the operations:

[tex]1.13\frac{m}{s^2}=a[/tex]

Now, we substitute the values in the equation for the coefficient of friction:

[tex]\mu=\frac{(12kg)(1.13\frac{m}{s^2})-(7kg)(9.8\frac{m}{s^2})+(7kg)(1.13\frac{m}{s^2})}{-(12kg)(9.8\frac{m}{s^2})}[/tex]

Solving the operations we get:

[tex]0.4=\mu[/tex]

Therefore, the coefficient of friction is 0.4

explain how the intensity of the UV light vaires across the Earth

Answers

Some factors determine the amount of UV radiation that reach certain part of Earth's surface. They are listed and briefly explained below:

• Cloud coverage. Water molecules on clouds scatter the radiation, hence the more clouds the less UV radiation.

,

• Ozone. Similarly as cloud coverage, the more concentration of ozone the less UV radiation that reaches the surface of the Earth.

,

• Angle of incidence. If the angle of incidence of the UV light is oblique the light will spread in a wider area, and hence the intensity is spread across this area.

,

• Aerosols. The molecules of aerosols also scatter the UV light.

,

• Elevation. The more the elevation the greater the amount of UV light.

You wish to lift a 720N crate of bricks to the 3rd floor of a building in a construction site. The 3rd floor is 16m high. How much work will that require?

Answers

ANSWER

[tex]11,520J[/tex]

EXPLANATION

Parameters given:

Weight (force), F = 720N

Height (distance), d = 16m

To find the work required to lift the crate to the 3rd floor, we have to find the product of the force (weight of the crate) and the distance it will be lifted.

Therefore, we have:

[tex]\begin{gathered} W=F\cdot d \\ W=720\cdot16 \\ W=11,520J \end{gathered}[/tex]

That is the work that it will require.

I think this is all statements are true but I just want to make sure

Answers

Given that a bug flies into the windshield of a car going. Let's select the correct statements.

According the NEwton's third law, the force exerted on the bug by the car is equal to the force extered on the car by the bug.

To determine the acceleration, we have:

[tex]a=\frac{F}{m}[/tex]

Where:

F is the force

m is the mass

The mass of the car will be greater than the mass of the bug.

Since the mass of the car is greater than the mass of the bug and they have the same force, we can say the acceleration of the bug is greater than the acceleration of the car.

Statement B is correct.

The force of impact is the same for both according to Newton's third Law.

Both the car and the bug deliver the same magnitude of impulse on each other.

Therefore, all statements are correct.

ANSWER:

All statements are true.

A wheel Was spinning at 2.8 rad/s. It took 3.2 seconds to stop completely. What is the acceleration of the wheel?

Answers

Given

The angular velocity is

[tex]\omega=2.8\text{ rad/s}[/tex]

The time taken,

[tex]t=3.2s[/tex]

To find

The acceleration of the wheel

Explanation

The acceleration is

[tex]\begin{gathered} \alpha=\frac{\omega}{t} \\ \Rightarrow\alpha=\frac{2.8}{3.2}=\frac{0.875rad}{s^2} \end{gathered}[/tex]

Conclusion

The acceleration is

[tex]0.875\text{ rad/s}^2[/tex]

A massless scaffold is held up by a wire at each end. The scaffold is 12 m long. A300-N box sits 4.0 m from the left end. What is the tension in each wire?1) left wire = 100 N; right wire = 200 N2) left wire = 200 N; right wire = 100 N3) left wire = 900 N; right wire = 2700 N4) left wire = 2700 N; right wire = 900 N

Answers

Free body diagram:

Given data:

Length of massless scaffold (end to end) L=12 m.

Weight of box m=300 N.

Length of massless scaffold (center to end) l=6 m.

As, the box sits 4.0 m from the left end, the distance of the box from the center of massless scaffold is given as,

[tex]\begin{gathered} r=6.0\text{ m}-4.0\text{ m} \\ =2.0\text{ m} \end{gathered}[/tex]

Balancing force in y direction,

[tex]T_1+T_2=300\text{ N}\ldots(1)[/tex]

The torque is given as,

[tex]\tau=perpendicular\text{ distance}\times force[/tex]

Therefore, torque along the center of massless scaffold is given as,

[tex]\begin{gathered} \Sigma\tau=0 \\ l\times T_1+r\times(300\text{ N})-l\times\tau_2=0 \\ 6\times T_1+2\times(300\text{ N})-6\times T_2=0 \\ 6T_1+600\text{ N}-6T_2=0 \\ 6(T_1+100\text{ N}-T_2)=0 \\ T_1+100\text{ N}-T_2=0 \\ T_1-T_2=-100\text{ N}\ldots(2) \end{gathered}[/tex]

Adding equation (1) and (2),

[tex]\begin{gathered} (T_1+T_2)+(T_1-T_2)=300\text{ N}-100\text{ N} \\ T_1+T_1+T_2-T_2=200\text{ N} \\ 2T_1=200\text{ N} \\ T_1=\frac{200\text{ N}}{2} \\ T_1=100\text{ N} \end{gathered}[/tex]

Substituting T1 in equation (1) we get,

[tex]\begin{gathered} 100\text{ N}+T_2=300\text{ N} \\ T_2=300\text{ N}-100\text{ N} \\ T_2=200\text{ N} \end{gathered}[/tex]

Therefore, tension in left wire is 100 N and tension on right wire is 200 N. Hence, option (1) is the correct choice.

Two 4.952 cm by 4.952 cm plates that form a parallel-plate capacitor are charged to +/- 0.576 nC. What is the electric field strength inside the capacitor if the spacing between the plates is 2.97 mm?

Answers

In order to solve this question, we will need to know the E-field of a capacitor

E = sigma/epsilon naught

To find sigma (charge density), we will need to divide the total charge by the area

sigma = (.576 x 10^-9)/(4.952x10^-2)^2 = 2.34 x 10^-7

E = 2.34 x 10^-7/ (8.85x10^-12) = 26541.04 V/m

Other Questions
write log_4 10 as a quotient of natural logarithms.ln__ln__ The graph below shows the solution to which system of inequalities?A. x< 1 and y2 xB. ys 1 and y>xC. x 1 and y> xD. y< 1 and y2 in the neck region , name the structure that plays a role in the immune system?a. thymusb. thyroid c. spleend. soft palate 8. The chart below shows the student lunch menu at a school. A lunch consists of onesandwich, one snack, and one drink.Lunch MenuSandwichSnack Drinkturkey apple juicebologna bananamilkpeanut buttercookieshamyogurtHow many different lunch choices does a student have?hirtants and subite or what is the slope and the y-intetercept of each problemy= -2x - 3y= -2x + 2 Out of 167 randomly selected adults in the United States who were surveyed, 70 exercise on a regular basis. Construct a 90% confidence interval for the proportion of all adults in the United States who exercise on a regular basis. Round to three decimal places Write the equation of the line that passes through the points `(1,\ -2)` and `(4,\ 4)`. A telephone plan consists of a monthly fee of $15 plus 20 cents per minute for long distance calls are made. Write an equation for the total monthly cost C, when n minutes are used for long distance calls. please help with this i will attach photo of figure Help n k oh hi k. Hi hold kb b g g I laws of exponent : multiplication and power to a powerquestion 2)))-2r 6r penelope had 1/2 of her birthday cake leftover so she decided to take it to work. she shared her birthday cake equally among herself and four of her co workers. what fraction of the cake did each person get? a. Determine whether the equation x/4-x/3=1 is a linear equation. If yes, identify the equation in standard form. Consider this system of linear equations:y = 4/5x - 3y = 4/5x + 1Try solving the system of equations algebraically and describe the result you get. Most environmental lawsuits are ____. a. civil suits b. criminal suits c. administrative suits d. statutory suits e. regulatory suits Because there are 3 feet in every yard, the formula F = 3 Y will convert Y yards into F feet. Find F ifY = 5 yards5 yards converts to ? feet. discuss the primary causes of bank failures in the united states during the late 1920s and early 1930s. Was the Kansas-Nebraska Act good for America? Why? How come the vertebrae protects the spinal but the PNS has no bone protection? A rectangular piece of metal has an area of 112 square centimeters. Its perimeter is 46 centimeters. What are the dimensions of the piece?